Company president: Our consultants report that, in general, the most efficient managers have excellent time manageme...

JayDee8732 on October 16, 2017

Help

How does A weaken?

Reply
Create a free account to read and take part in forum discussions.

Already have an account? log in

Mehran on October 16, 2017

Hi @JayDee8732, thanks for your post. This is a Bizarro Weaken question. This means that four of the answer choices weaken the argument presented in the stimulus; the fifth (and correct) answer choice has no impact or may strengthen the argument in the stimulus.

Let's first, as always, review the stimulus carefully.

The company president makes an argument, as follows:
P: Our consultants report that, in general, the most efficient managers have excellent time management.
C: So, to improve productivity, I recommend that we make a seminar on time management techniques available to our middle-level managers.

All right. Notice that the president leaps from a premise that establishes a correlation (between efficiency and time management skills) to a conclusion that assumes that one thing (time management) will cause the other (improved efficiency/productivity). This is problematic reasoning.

Now let's examine answer choice (A).

This answer choice clarifies something important about the consultants' methodology. Specifically, it says that the consultants basically look at efficiency and time management the same way. If this is true, then the fact that a manager is deemed by a consultant to be "most efficient" does not really tell us anything independently about that manager's time management skills in the sense that the company president assumes. Put differently, the company president's assumption that improved time management would cause improved efficiency / productivity is even more faulty if the consultants basically think of efficiency and time management as interchangeable attributes. If (A) is true, then the premise is even less supportive of the conclusion than we thought before. This answer choice, therefore, does weaken the argument in the stimulus. Since this is a Bizarro Weaken question, we can eliminate it.

Hope this helps!